LSAT and Law School Admissions Forum

Get expert LSAT preparation and law school admissions advice from PowerScore Test Preparation.

User avatar
 Dave Killoran
PowerScore Staff
  • PowerScore Staff
  • Posts: 5853
  • Joined: Mar 25, 2011
|
#44073
Complete Question Explanation
(The complete setup for this game can be found here: lsat/viewtopic.php?t=6217)

The correct answer choice is (C)

If V is one of three stores that carry R, then those three stores must be T, X, and V. So S must carry J, O, and F. And Z must carry O, F, or both.

Answer choice (A) is incorrect because S and Z must have at least O or F in common.

Answer choice (B) is incorrect because V could carry only R.

Answer choice (C) is the correct answer choice. S and Z have at least O or F in common (possibly both).

Answer choice (D) is incorrect because Z could carry only F.

Answer choice (E) is incorrect because V could carry only R.

Get the most out of your LSAT Prep Plus subscription.

Analyze and track your performance with our Testing and Analytics Package.